A Few Personal Favorites from CMIMC (Part II)
by djmathman, Jan 30, 2017, 2:51 AM
Once again, here are a few of my favorite problems from this year's contest that I wrote along with their solutions. I think I wrote 29 questions out of the 75 that appeared on the test, which means I've broken 60 total problems over the two years; I'm not sure if this is sustainable....
Solution
Remark
Solution
Solution
Solution
Solution (jointly by me and Evan Chen)
Solution
Remark
Oops this list doesn't have much variety....
2017 CMIMC Team #2 wrote:
Suppose
,
, and
are nonzero complex numbers such that
. Compute ![\[(x+y+z)\left(\dfrac1x+\dfrac1y+\dfrac1z\right).\]](//latex.artofproblemsolving.com/b/4/e/b4e212f566995443c545f40f7fc67ddeae14e83e.png)




![\[(x+y+z)\left(\dfrac1x+\dfrac1y+\dfrac1z\right).\]](http://latex.artofproblemsolving.com/b/4/e/b4e212f566995443c545f40f7fc67ddeae14e83e.png)
Solution
Let
,
, and
. Note that
and
Thus the condition becomes
Hence
, and so dividing both sides by
gives the desired answer of
.




![\[x^3+y^3+z^3 = (x+y+z)(x^2+y^2+z^2-xy-yz-zx) + 3xyz = S_1^3-3S_1S_2 + 3S_3.\]](http://latex.artofproblemsolving.com/b/2/c/b2c3c4ac288e1df9aa8743086efe4b1edee8cfd1.png)
![\[S_1(S_1^2-2S_2) = S_1^3 - 3S_1S_2 + 3S_3\quad\implies\quad S_1S_2 = 3S_3.\]](http://latex.artofproblemsolving.com/a/5/e/a5e32710c5ffb7d22938aec3ecc22b37683b03da.png)



Remark
The only reason this is so early on the test is because it is very hackable (say by chosing
and solving for
).


2017 CMIMC Geometry #3 wrote:
In acute triangle
, points
and
are the feet of the angle bisector and altitude from
respectively. Suppose that
and
. Compute
.







Solution
Let
and
. Note that by Angle Bisector Theorem,
Thus
and
. Now note that by Pythagorean Theorem,
. This means that
Simplification yields
.


![\[\dfrac{DC}{AC} = \dfrac{DB}{AB} = \dfrac{DC-DB}{AC-AB} = \dfrac23.\]](http://latex.artofproblemsolving.com/f/8/7/f8789745e9ecd789522c1840db2df4fff791fdd0.png)



![\[(EC-EB)(EC+EB) = (AC-AB)(AC+AB)\quad\implies\quad (EC-EB)\cdot\dfrac23(x+y) = 36(x+y).\]](http://latex.artofproblemsolving.com/e/d/a/eda72779f000e360453681628a443758df673854.png)

2017 CMIMC Algebra #7 wrote:
Let
,
, and
be complex numbers satisfying the system of equations
Find
.





Solution
Let
for all nonnegative integers
. Note that
This is this identity that will be the workhorse for our solution.
Note that plugging in
gives
, or
. Similarly, note that the
case gives
. Next, the
case yields
, and so
. Now write
Finally, recall that
and
implies
, so ![\[11(5) - 16 = 39 = 3abc\quad\implies\quad abc=\boxed{13}.\]](//latex.artofproblemsolving.com/a/b/9/ab9189e81812eed802a73679ae35c1e26e2876ee.png)
![\[E_r = \dfrac{a^r}{b+c}+\dfrac{b^r}{c+a}+\dfrac{c^r}{a+b}\]](http://latex.artofproblemsolving.com/2/e/4/2e4b30f560e3cb38e37db1dc3c870fc6ddd11ca6.png)


Note that plugging in












![\[11(5) - 16 = 39 = 3abc\quad\implies\quad abc=\boxed{13}.\]](http://latex.artofproblemsolving.com/a/b/9/ab9189e81812eed802a73679ae35c1e26e2876ee.png)
2017 CMIMC Geometry #9 wrote:
Let
be an acute triangle with circumcenter
, and let
denote the point on
for which
. The circumcircle of
intersects lines
and
for the second time at
and
respectively. Suppose that
,
, and
are concurrent. If
and
, compute
.
















Solution
First remark that
is antiparallel to
, so
.
![[asy]
import olympiad;
size(250);
defaultpen(linewidth(0.8));
pair A = dir(119.5), B = dir(215), C = dir(325), Q = (A.x,-1*A.y), P = intersectionpoint(A--Q,B--C), X = 2*B - A, O = origin, Y = circumcenter(B,O,C);
path cir = circumcircle(B,O,C);
pair[] Dp = intersectionpoints(A--C, cir), Ep = intersectionpoints(A--X,cir);
pair D = Dp[0], E = Ep[1];
draw(B--C--A--E--D^^circumcircle(A,B,C)^^A--Q);
draw(arc(Y,circumradius(B,O,C),-5,185));
draw(rightanglemark(A,P,B,1.5)^^E--O--D);
dot(O);
draw(circumcircle(A,D,E),linetype("4 4"));
label("$A$",A,dir(A));
label("$B$",B,dir(190));
label("$C$",C,dir(350));
label("$Q$",Q,dir(Q));
label("$D$",D,dir(40));
label("$E$",E,dir(220));
label("$O$",O,N);
label("$P$",P,SE);
[/asy]](//latex.artofproblemsolving.com/e/a/6/ea63bb8060a3a3f32e8be1f76563757747546980.png)
Let
be the foot of the perpendicular from
to
. Note that
is the radical axis of
and
and that
is the radical axis of
and
. Hence
is the radical center of all three circles, meaning that
is the radical axis of
and
. Since
is a chord of
, we may deduce that
is cyclic.
Furthermore, a simple angle chase reveals that
which implies
. Similarly
, so
is the orthocenter of
. This means that
and
are isogonal with respect to
. As a result,
is a diameter of
, which implies that
is a parallelogram. This means that
But note that if
is the circumradius of
, then
which we know is equal to
since
is a diameter of
. Thus ![\[AQ = \sqrt{2(OD^2+OE^2)} = \sqrt{2(3^2+7^2)} = \boxed{2\sqrt{29}}.\]](//latex.artofproblemsolving.com/d/7/8/d78ac74f51234bc38f7a10bcdca2bfde11175211.png)



![[asy]
import olympiad;
size(250);
defaultpen(linewidth(0.8));
pair A = dir(119.5), B = dir(215), C = dir(325), Q = (A.x,-1*A.y), P = intersectionpoint(A--Q,B--C), X = 2*B - A, O = origin, Y = circumcenter(B,O,C);
path cir = circumcircle(B,O,C);
pair[] Dp = intersectionpoints(A--C, cir), Ep = intersectionpoints(A--X,cir);
pair D = Dp[0], E = Ep[1];
draw(B--C--A--E--D^^circumcircle(A,B,C)^^A--Q);
draw(arc(Y,circumradius(B,O,C),-5,185));
draw(rightanglemark(A,P,B,1.5)^^E--O--D);
dot(O);
draw(circumcircle(A,D,E),linetype("4 4"));
label("$A$",A,dir(A));
label("$B$",B,dir(190));
label("$C$",C,dir(350));
label("$Q$",Q,dir(Q));
label("$D$",D,dir(40));
label("$E$",E,dir(220));
label("$O$",O,N);
label("$P$",P,SE);
[/asy]](http://latex.artofproblemsolving.com/e/a/6/ea63bb8060a3a3f32e8be1f76563757747546980.png)
Let
















Furthermore, a simple angle chase reveals that
![\[\angle ADO = \angle OBC = 90^\circ-\angle A,\]](http://latex.artofproblemsolving.com/2/e/8/2e801448ce87bfa304658ba23c434e3d0a16c483.png)










![\[2(OD^2+OE^2) = OQ^2+DE^2 = OA^2+DE^2.\]](http://latex.artofproblemsolving.com/e/1/6/e1685f08a93ea433daa5ef889018e0b893f8db66.png)


![\[OA^2+DE^2 = (2R'\cos A)^2 + (2R'\sin A)^2 = 4R'^2,\]](http://latex.artofproblemsolving.com/f/9/5/f95a9be841012c64062c81879fcaeb26ad219407.png)



![\[AQ = \sqrt{2(OD^2+OE^2)} = \sqrt{2(3^2+7^2)} = \boxed{2\sqrt{29}}.\]](http://latex.artofproblemsolving.com/d/7/8/d78ac74f51234bc38f7a10bcdca2bfde11175211.png)
2017 CMIMC Team #9 wrote:
Circles
and
are externally tangent to each other. Circle
is placed such that
is internally tangent to
at
while
is internally tangent to
at
. Line
is tangent to
at
and
at
and furthermore intersects
at points
and
with
. Suppose that
,
, and
. Compute the length of line segment
.






















Solution (jointly by me and Evan Chen)
First, recall by homothety that
is the midpoint of minor arc
. This means that
whence
is a cyclic quadrilateral. Now
is the radical center of
,
, and
, so in particular if
, then
is the common internal tangent of the two circles.
![[asy]
import olympiad;
size(270);
defaultpen(linewidth(0.8)+fontsize(11pt));
real r1 = 2.42, r2 = 4.56, d = sqrt(r1*r2);
pair O1 = (-1*d,r1), O2 = (d,r2), P = (-d,0), Q = (d,0);
path circ1 = circle(O1,r1), circ2 = circle(O2,r2);
draw(circ1^^circ2);
pair M = (1.34, -4.16), pt1 = 3*P - 2*M, pt2 = 3*Q - 2*M;
pair[] X = intersectionpoints(P--pt1, circ1), Y = intersectionpoints(Q--pt2, circ2);
path omega = circumcircle(M,X[1], Y[1]);
pair O = circumcenter(M,X[1],Y[1]);
draw(omega);
pair Pext = 2*P-Q, Qext = 2*Q-P;
pair[] end = intersectionpoints(Pext--Qext, omega);
pair A = end[0], B = end[1];
draw(Y[1]--X[1]^^A--B);
draw(X[1]--M--Y[1]^^O1--O2, linetype("3 3"));
pair Os = circumcenter(P,Q,X[1]);
real r = circumradius(P,Q,X[1]);
draw(circle(Os,r), linetype("4 4"));
pair T = foot(M,O1,O2);
dot(O1^^O2^^P^^Q^^M^^T^^X[1]^^Y[1]);
draw(M--T, linetype("3 3"));
label("$A$",A,1.2*dir(235));
label("$B$",B,1.2*dir(305));
label("$X$",X[1],2*dir(165));
label("$Y$",Y[1],1.5*dir(35));
label("$P$",P,1.5*dir(250));
label("$Q$",Q,1.5*dir(290));
label("$T$",T,2*dir(330));
label("$M$",M,1.5*S);
[/asy]](//latex.artofproblemsolving.com/6/c/a/6cac7490e8b5ed011e167bbd0264f0e506610f26.png)
\par Set
, and define
. Note that a bit of angle chasing yields
, so
Thus
and
. Now Stewart's Theorem on
with cevian
gives us the value of
directly, by
Finally, if we let
and
now, we have
, and similarly
. Thus
so
and
and
.


![\[\angle XPA = \dfrac{\widehat{AX}+\widehat{MB}}2 = \dfrac{\widehat{AX}+\widehat{MA}}2 = \angle XYQ,\]](http://latex.artofproblemsolving.com/c/6/9/c694ce449d1cbac697f43ad0c913b07cd7771535.png)







![[asy]
import olympiad;
size(270);
defaultpen(linewidth(0.8)+fontsize(11pt));
real r1 = 2.42, r2 = 4.56, d = sqrt(r1*r2);
pair O1 = (-1*d,r1), O2 = (d,r2), P = (-d,0), Q = (d,0);
path circ1 = circle(O1,r1), circ2 = circle(O2,r2);
draw(circ1^^circ2);
pair M = (1.34, -4.16), pt1 = 3*P - 2*M, pt2 = 3*Q - 2*M;
pair[] X = intersectionpoints(P--pt1, circ1), Y = intersectionpoints(Q--pt2, circ2);
path omega = circumcircle(M,X[1], Y[1]);
pair O = circumcenter(M,X[1],Y[1]);
draw(omega);
pair Pext = 2*P-Q, Qext = 2*Q-P;
pair[] end = intersectionpoints(Pext--Qext, omega);
pair A = end[0], B = end[1];
draw(Y[1]--X[1]^^A--B);
draw(X[1]--M--Y[1]^^O1--O2, linetype("3 3"));
pair Os = circumcenter(P,Q,X[1]);
real r = circumradius(P,Q,X[1]);
draw(circle(Os,r), linetype("4 4"));
pair T = foot(M,O1,O2);
dot(O1^^O2^^P^^Q^^M^^T^^X[1]^^Y[1]);
draw(M--T, linetype("3 3"));
label("$A$",A,1.2*dir(235));
label("$B$",B,1.2*dir(305));
label("$X$",X[1],2*dir(165));
label("$Y$",Y[1],1.5*dir(35));
label("$P$",P,1.5*dir(250));
label("$Q$",Q,1.5*dir(290));
label("$T$",T,2*dir(330));
label("$M$",M,1.5*S);
[/asy]](http://latex.artofproblemsolving.com/6/c/a/6cac7490e8b5ed011e167bbd0264f0e506610f26.png)
\par Set



![\[MA^2 = MP\cdot MX = MT^2\quad\implies\quad MT = r.\]](http://latex.artofproblemsolving.com/4/9/5/49519387ecfd5c85f91fa7378882008a22d63cf4.png)





![\[
4 \cdot 5 \cdot 9 + 9(r-2)^2 = 4r^2 + 5r^2
\implies r = 6.
\]](http://latex.artofproblemsolving.com/9/6/0/960b6e6d606f1e882ed647fb357a3fc81cec6f10.png)




![\[ x\left(x+\dfrac{14}x\right) = y\left(y+\dfrac{18}y\right) = r^2 = 36, \]](http://latex.artofproblemsolving.com/4/9/0/49053afd2293a04524559986f4c6c7224c427728.png)



2017 CMIMC Algebra #10 wrote:
Let
denote the largest possible real number such that there exists a nonconstant polynomial
with
for all
. Compute the sum of all values of
over all nonconstant polynomials
satisfying the above constraint for this
.


![\[P(z^2)=P(z-c)P(z+c)\]](http://latex.artofproblemsolving.com/e/d/5/ed541828dbab4324c99422e8c114708bfda726c8.png)




We claim that
.
\par First note that if
is a root of
, then plugging in
yields
so that
is a root of
as well. Similarly,
must also be a root of
.
Now suppose
, and let
be a possible root of
. Define a sequence of complex numbers
such that
and such that
is either equal to
or
. I claim it is always possible to choose a sequence with the property that the sequence
is strictly increasing. To see this, recall by the Parallelogram Law,
It thus follows that one of
and
must be at least
(else the entire sum would be too small), so we can choose
such that
. But note that
which is always true for
. Thus
, as desired. It follows that
is an infinite sequence of roots of
, which is a contradiction.
It suffices to classify all polynomials satisfying the equation when
. To do this, remark that there are two equality cases in the above analysis. The first occurs in the choice of
; equality here occurs when
, or when
is purely imaginary. The second equality case occurs in completing the square. For
, we need
, i.e.
. It follows that
and
are the only possible roots of
, and furthermore it is easy to see that these roots must occur with equal multiplicity. Indeed, taking
, we see that
Hence
for some integer
, and it follows that the sum of all possible values of
is ![\[\sum_{n\geq 1}\left(\dfrac19+\frac14\right)^n = \sum_{n\geq 1}\left(\frac{13}{36}\right)^n = \boxed{\frac{13}{23}}.\]](//latex.artofproblemsolving.com/f/0/3/f03af8740fb3afa666f4cfc21f021382a060cc57.png)

\par First note that if



![\[P((\alpha+c)^2) = P(\alpha)P(\alpha+2c) = 0,\]](http://latex.artofproblemsolving.com/d/9/5/d958ed124092beb6d0cb1a6b96ae8847ca44ffa8.png)




Now suppose









![\[|z-c|^2+|z+c|^2 = 2(|z|^2+c^2).\]](http://latex.artofproblemsolving.com/e/8/0/e801773101553fabd74a73e7e12e330c4a0716eb.png)





![\[|z|^2+c^2>|z|\quad\iff\quad \left(|z|-\frac12\right)^2 + c^2 > \frac14,\]](http://latex.artofproblemsolving.com/2/d/7/2d7cf68eb0decb9c39ed5097a82a1ed04a90a9b3.png)




It suffices to classify all polynomials satisfying the equation when















![\[\sum_{n\geq 1}\left(\dfrac19+\frac14\right)^n = \sum_{n\geq 1}\left(\frac{13}{36}\right)^n = \boxed{\frac{13}{23}}.\]](http://latex.artofproblemsolving.com/f/0/3/f03af8740fb3afa666f4cfc21f021382a060cc57.png)
Remark
Darn this solution took me about two days to find, but I'm really glad I found it. Still don't know how to deal with
though....

Oops this list doesn't have much variety....
This post has been edited 3 times. Last edited by djmathman, Jan 30, 2017, 2:59 AM